2
$\begingroup$

$J$ is a symmetric matrix (built from 6j symbols...it's always knot theory in disguise when I ask :-), $D$ a diagonal matrix, and $B=DJ$. $S$ is a diagonal sign matrix (entries all $+1$ or $-1$). $I$ is the identity matrix. From a bit of diagram juggling, I find the determining equations for $B$ are simply:

$B^2=I$

$(BS)^3=I$

Which I then happily solve by brute force (actually some elements of $J$ are known beforehand) but already for a 6*6 matrix this leads to a big mess.
Is there a more "intelligent" method? (I still need to know all elements of $B$...Sidenote: If $S$ contains only one or two $-1$, I already know the closed form for any size of $B$.) Even better, if yes, would it still work if, in a more general version, $S$ is still diagonal but $S^2=I$ no longer holds?

$\endgroup$
4
  • 4
    $\begingroup$ Solve ... for what? $J$? $D$? $B$? $S$? $\endgroup$ Sep 11, 2013 at 12:57
  • 1
    $\begingroup$ @Gerry: If I were the OP, I would be tempted to answer "yes" ;-) $\endgroup$ Sep 12, 2013 at 0:06
  • $\begingroup$ @Gerry - your question is very justified in the general case, where no variables are known at all (and where I express B in terms of S elements - that's the best I can do). J and D are mostly irrelevant to this question (except they restrict the form of B somewhat). In this special case, I usually solve for B for any sign combination of S. $\endgroup$ Sep 12, 2013 at 14:32
  • $\begingroup$ @anyone who stumbles over this: It seems (I experimented with MATHEMATICA's NSOLVE command) that if S and D are given, B is uniquely (apart from signs and complex conjugates) defined. Of course I can't prove it. $\endgroup$ Sep 18, 2013 at 16:27

1 Answer 1

4
$\begingroup$

Assuming $S^2=I$, your other two relations give you a presentation for the symmetric group of degree 3. So you want to find linear representations of this group (satisfying the extra condition $B=DJ$). In particular, note that $B$ and $S$ have the same eigenvalues. Also, note that the number of -1's in $S$ gives you a constraint on the irreducible subconstituents of your representation.

$\endgroup$
0

Your Answer

By clicking “Post Your Answer”, you agree to our terms of service and acknowledge you have read our privacy policy.

Not the answer you're looking for? Browse other questions tagged or ask your own question.